Difference between revisions of "1997 PMWC Problems/Problem T10"

(Problem)
Line 3: Line 3:
  
 
==Solution==
 
==Solution==
Less than 9.
+
The answer is <math>\boxed{0}.</math>
{{solution}}
+
 
 +
We first seat <math>1.</math> Then we put all odd numbers in one way and all even numbers in another direction. The result is <math>12,10,8,6,4,2,1,3,5,7,9,11,12...</math> when we read the table. It satisfies the conditions and never has <math>4</math> as a difference.
  
 
==See Also==
 
==See Also==
  
 
{{PMWC box|year=1997|num-b=T9|after=Last<br />Problem}}
 
{{PMWC box|year=1997|num-b=T9|after=Last<br />Problem}}

Revision as of 17:05, 15 July 2018

Problem

The twelve integers $1, 2, 3,\dots, 12$ are arranged in a circle such that the difference of any two adjacent numbers is either $2, 3,$ or $4$. What is the maximum number of the difference $4$ can occur in any such arrangement?

Solution

The answer is $\boxed{0}.$

We first seat $1.$ Then we put all odd numbers in one way and all even numbers in another direction. The result is $12,10,8,6,4,2,1,3,5,7,9,11,12...$ when we read the table. It satisfies the conditions and never has $4$ as a difference.

See Also

1997 PMWC (Problems)
Preceded by
Problem T9
Followed by
Last
Problem
I: 1 2 3 4 5 6 7 8 9 10 11 12 13 14 15
T: 1 2 3 4 5 6 7 8 9 10